0
$\begingroup$

I am familiar with the expression for spin 1/2 but haven't seen one for spin 1.

$\endgroup$
4
  • 1
    $\begingroup$ Could you explain what you mean exactly? What is the expression you're familiar with for spin 1/2? $\endgroup$
    – fqq
    Jan 17, 2014 at 16:13
  • $\begingroup$ Using the same method as this uvm.edu/~kspartal/EPR_Simulation/AboutSpins.pdf which is for the spin 1/2 particle and apply it to the spin 1 particle $\endgroup$
    – user32462
    Jan 19, 2014 at 17:21
  • $\begingroup$ If you plug the spin-1 representation for $S_x$, $S_y$ and $S_z$ (which I posted below) into the very first equation of this pdf (i.e. into the definition of $S_r$) you can go through the steps of this document and obtain the respective expressions for spin-1. The difference being, that you are now dealing with $3\times 3$-matrices instead. $\endgroup$
    – André
    Jan 26, 2014 at 11:03
  • $\begingroup$ Already done it. The problem just came when normalising my eigenvectors using the mathematica function I had some degeneracies but calculating it by hand sorted it out. $\endgroup$
    – user32462
    Jan 26, 2014 at 13:31

1 Answer 1

1
$\begingroup$

I'm not entirely sure if I'm getting your question correctly, but the general spin operator is given by \begin{equation} \widehat{S} = a\cdot \widehat{S}_x + b\cdot \widehat{S}_y + c\cdot \widehat{S}_z \end{equation} where $a,b,c \in \mathbb{R}$ such that $\vec{n} \equiv (a\; b\; c)$ is the direction in which you measure the spin. Note that so far, no particular spin (1/2, 1, 3/2 etc.) has been specified. As you will probably know, the operators $\widehat{S}_x, \widehat{S}_y$ and $\widehat{S}_z$ are required to satisfy a very specific algebraic relation, namely \begin{equation} [\widehat{S}_\alpha, \widehat{S}_\beta] = i\hbar\ \varepsilon_{\alpha\beta}^{~~~~~\gamma}\ \widehat{S}_\gamma \qquad \forall\, \alpha,\beta,\gamma \in \{x,y,z\}\ . \end{equation} Now, to put it in "simple words", you go looking for matrices that satisfy this relation (i.e. representations of the algebra). If you look for $2\times 2$-matrices you may find that \begin{equation} \widehat{S}_x \stackrel{\cdot}{=} \frac{\hbar}{2}\begin{pmatrix}0 & 1 \\ 1 & 0 \end{pmatrix},\ \widehat{S}_y \stackrel{\cdot}{=} \frac{\hbar}{2}\begin{pmatrix}0 & -i \\ i & 0 \end{pmatrix},\ \widehat{S}_z \stackrel{\cdot}{=} \frac{\hbar}{2}\begin{pmatrix}1 & 0 \\ 0 & -1 \end{pmatrix}\ \end{equation} will do the trick (recognize the Pauli matrices). This representation by $2\times 2$ matrices is called a spin-1/2 representation. Now, you may go on and look for a representation by $3\times 3$ matrices and find \begin{equation} \widehat{S}_x \stackrel{\cdot}{=} \frac{\hbar}{\sqrt{2}}\begin{pmatrix}0 & 1 & 0 \\ 1 & 0 & 1 \\ 0 & 1 & 0 \end{pmatrix}\, ,\ \widehat{S}_y \stackrel{\cdot}{=} \frac{\hbar}{\sqrt{2}}\begin{pmatrix}0 & -i & 0 \\ i & 0 & -i \\ 0 & i & 0 \end{pmatrix}\, ,\ \widehat{S}_z \stackrel{\cdot}{=} \hbar\begin{pmatrix}1 & 0 & 0 \\ 0 & 0 & 0 \\ 0 & 0 & -1 \end{pmatrix}\ . \end{equation} This is a spin-1 representation. Note that conventionally the basis is chosen in such a way that $\widehat{S}_z$ is a diagonal matrix and the entries on its main diagonal correspond to the possible outcomes of your measurements, i.e. ($+\frac{\hbar}{2}$, $-\frac{\hbar}{2}$) for the spin-1/2 case and ($+\hbar$, $0$, $-\hbar$) for the spin-1 case.

$\endgroup$
3
  • 3
    $\begingroup$ This is fine for non-elementary particles, as well as for atoms with angular momentum $L$, but beware of photons, which don't have a rest frame, and therefore only helicity. $\endgroup$ Jan 17, 2014 at 17:03
  • $\begingroup$ That is absolutely true. As I said, I'm not sure if I'm getting the question correctly. $\endgroup$
    – André
    Jan 17, 2014 at 17:09
  • $\begingroup$ My best interpretation of the question is the literal one, so I would say a Stern-Gerlach apparatus, heh $\endgroup$ Jan 18, 2014 at 11:29

Your Answer

By clicking “Post Your Answer”, you agree to our terms of service and acknowledge you have read our privacy policy.

Not the answer you're looking for? Browse other questions tagged or ask your own question.